www.vorhilfe.de
- Förderverein -
Der Förderverein.

Gemeinnütziger Verein zur Finanzierung des Projekts Vorhilfe.de.
Hallo Gast!einloggen | registrieren ]
Startseite · Mitglieder · Impressum
Forenbaum
^ Forenbaum
Status VH e.V.
  Status Vereinsforum

Gezeigt werden alle Foren bis zur Tiefe 2

Navigation
 Startseite...
 Suchen
 Impressum
Das Projekt
Server und Internetanbindung werden durch Spenden finanziert.
Organisiert wird das Projekt von unserem Koordinatorenteam.
Hunderte Mitglieder helfen ehrenamtlich in unseren moderierten Foren.
Anbieter der Seite ist der gemeinnützige Verein "Vorhilfe.de e.V.".
Partnerseiten
Weitere Fächer:

Open Source FunktionenplotterFunkyPlot: Kostenloser und quelloffener Funktionenplotter für Linux und andere Betriebssysteme
Forum "Uni-Stochastik" - Gesetz der großen Zahl
Gesetz der großen Zahl < Stochastik < Hochschule < Mathe < Vorhilfe
Ansicht: [ geschachtelt ] | ^ Forum "Uni-Stochastik"  | ^^ Alle Foren  | ^ Forenbaum  | Materialien

Gesetz der großen Zahl: Tipp, Rückfrage, Korrektur
Status: (Frage) beantwortet Status 
Datum: 00:10 Mo 02.04.2012
Autor: Mija

Aufgabe
Sei $sup [mm] E(X_{n}^2) [/mm] < [mm] \infty$, $(X_n)$ [/mm] unabhängig, [mm] $\exists [/mm] N: [mm] E(X_n)=\mu$ [/mm] für $n>N$

z.z.: [mm] $\bruch{1}{n} \summe_{i=1}^{n} X_k$ [/mm] konvergiert fast sicher. Gegen welchen Grenzwert?


Hallo, ich habe die obenstehende Aufgabe gelöst, allerdings bin ich mir unsicher, ob das wirklich alles stimmt, was ich hier stehen habe bzw. ob ich auch alles habe.

Es ist ja gegeben, dass die [mm] $X_n$ [/mm] unabhängig sind. Daraus folgt erstmal, dass sie unkorreliert sind, d.h. [mm] $Cov(X_m,X_n)=0$. [/mm]
Also

[mm] $Cov(X_m,X_n)=0 \gdw [/mm] 0= [mm] E(X_m [/mm] * [mm] X_n) [/mm] - [mm] \underbrace{E(X_m)}_{=\mu_m}*\underbrace{E(X_n)}_{=\mu_n}$ [/mm]

[mm] $\Rightarrow E(X_m X_n) [/mm] = [mm] \mu_m [/mm] * [mm] \mu_n$ [/mm]
[mm] $\Rightarrow E(X_{n}^2)=\mu_{n}^2$ [/mm]
[mm] $Var(X_n)= E(X_{n}^2) [/mm] - [mm] E(X_n)^2 [/mm] = [mm] \mu_{n}^2 [/mm] - [mm] \mu_{n}^2 [/mm] = 0 < [mm] \infty$ [/mm]

Aus dem starken Gesetz der großen Zahl folgt dann
[mm] $\bruch{1}{n} \summe_{i=1}^{n} X_k \to \mu$ $\IP$ [/mm] - fast sicher, da  [mm] $E(X_i) [/mm] =  [mm] E(X_1)$ [/mm]

Ich würde mich freuen, wenn sich das jemand angucken könnte! :)

        
Bezug
Gesetz der großen Zahl: Antwort
Status: (Antwort) fertig Status 
Datum: 09:34 Mo 02.04.2012
Autor: tobit09

Hallo Mija,

> Es ist ja gegeben, dass die [mm]X_n[/mm] unabhängig sind. Daraus
> folgt erstmal, dass sie unkorreliert sind, d.h.
> [mm]Cov(X_m,X_n)=0[/mm].

Für [mm] $m\not=n$. [/mm]

>  Also
>  
> [mm]Cov(X_m,X_n)=0 \gdw 0= E(X_m * X_n) - \underbrace{E(X_m)}_{=\mu_m}*\underbrace{E(X_n)}_{=\mu_n}[/mm]
>  
> [mm]\Rightarrow E(X_m X_n) = \mu_m * \mu_n[/mm]

Immernoch nur für [mm] $m\not=n$. [/mm]

>  [mm]\Rightarrow E(X_{n}^2)=\mu_{n}^2[/mm]

Diese Schlussfolgerung stimmt dann nicht.

> [mm]Var(X_n)= E(X_{n}^2) - E(X_n)^2 = \mu_{n}^2 - \mu_{n}^2 = 0 < \infty[/mm]

[mm] $Var(X_n)=0$ [/mm] hieße [mm] $X_n$ [/mm] fast sicher konstant. Das muss hier sicherlich nicht gelten.
  

> Aus dem starken Gesetz der großen Zahl folgt dann
>  [mm]\bruch{1}{n} \summe_{i=1}^{n} X_k \to \mu[/mm] [mm]\IP[/mm] - fast
> sicher, da  [mm]E(X_i) = E(X_1)[/mm]

Warum sollte [mm] $E(X_i)=E(X_1)$ [/mm] gelten? Wir haben [mm] $E(X_i)=\mu$ [/mm] ja nur für große $i$ und nicht für alle $i$.


Leider habe ich gerade keine Zeit für weitere Hinweise, daher lasse ich die Frage auf teilweise beantwortet.

Bitte poste doch mal eure genaue Formulierung des Gesetzes der großen Zahlen in der stärksten Form, die ihr kennengelernt habt.


Viele Grüße
Tobias

Bezug
                
Bezug
Gesetz der großen Zahl: Mitteilung
Status: (Mitteilung) Reaktion unnötig Status 
Datum: 10:33 Mo 02.04.2012
Autor: Mija

Das starke Gesetz der großen Zahl haben wir in der Vorlesung so formuliert:

Sei [mm] $(X_n)_{n \in \IN}$ [/mm] Folge von Zufallsvariablen, unkorreliert, in [mm] $\mathcal{L}^2{(\IP)} [/mm] mit [mm] $Var(X_n) \le [/mm] c < n, n [mm] \in \IN$. [/mm]
Dann gilt: [mm] $\bruch{1}{n} \summe_{i=1}^{n} (X_i [/mm] - [mm] E(X_i)) \to [/mm] 0$ [mm] $\IP$-fast [/mm] sicher.

Bezug
        
Bezug
Gesetz der großen Zahl: Antwort
Status: (Antwort) fertig Status 
Datum: 15:57 Mo 02.04.2012
Autor: tobit09


> Sei [mm]sup E(X_{n}^2) < \infty[/mm], [mm](X_n)[/mm] unabhängig, [mm]\exists N: E(X_n)=\mu[/mm]
> für [mm]n>N[/mm]
>  
> z.z.: [mm]\bruch{1}{n} \summe_{i=1}^{n} X_k[/mm] konvergiert fast
> sicher. Gegen welchen Grenzwert?

In der Summe soll es übrigens sicherlich [mm] $X_i$ [/mm] statt [mm] $X_k$ [/mm] heißen.


1. Überlege dir, dass die Voraussetzungen des Gesetzes der großen Zahlen, wie du es in deiner Mitteilung gepostet hast, für unsere Folge der [mm] $X_n$ [/mm] erfüllt ist.

Deshalb konvergiert die Folge der

     [mm] $Y_n:=\bruch1n\summe_{i=1}^n(X_i-E(X_i))$ [/mm]

fast sicher gegen 0.

Es gilt

     [mm] $Y_n=\bruch1n\summe_{i=1}^nX_i-\underbrace{\bruch1n\summe_{i=1}^nE(X_i)}_{=:a_n}$. [/mm]

2. Für $n>N$ gilt

     [mm] $a_n=\bruch1n\summe_{i=1}^NE(X_i)+\bruch1n\summe_{i=N+1}^nE(X_i)=\ldots$. [/mm]

3. Somit konvergiert die Folge der [mm] $a_n$ [/mm] gegen ...

4. Somit konvergiert

     [mm] $\bruch1n\summe_{i=1}^nX_i=Y_n+a_n$ [/mm]

fast sicher gegen ...

Bezug
                
Bezug
Gesetz der großen Zahl: Mitteilung
Status: (Mitteilung) Reaktion unnötig Status 
Datum: 19:02 Mo 02.04.2012
Autor: Mija

Aah, jetzt hab ich es verstanden :)

Also unsere Folge [mm] $a_n$ [/mm] konvergiert gegen [mm] $\mu$. [/mm]
Also gilt [mm] $\bruch{1}{n} X_k \to \mu$ $\IP$-fast [/mm] sicher.

Bezug
Ansicht: [ geschachtelt ] | ^ Forum "Uni-Stochastik"  | ^^ Alle Foren  | ^ Forenbaum  | Materialien


^ Seitenanfang ^
ev.vorhilfe.de
[ Startseite | Mitglieder | Impressum ]